Step Function Laplace w/ piecewise

Click For Summary
SUMMARY

The discussion focuses on solving the differential equation y'' + 2y' + 2y = h(t) using the Laplace transform, where h(t) is defined piecewise. The solution involves taking the Laplace transform of both sides, leading to the equation (s^2 + 2s + 2)F(s) - 1 = \mathcal{L}(h(t)). The user expresses difficulty in simplifying the resulting expression for F(s), which is ultimately resolved by recognizing the inverse Laplace transforms of the functions 1/D_k(s) for k=0, 1, 2. The final solution is expressed in terms of these transforms and the Heaviside step function.

PREREQUISITES
  • Understanding of Laplace transforms and their properties
  • Familiarity with solving second-order linear differential equations
  • Knowledge of piecewise functions and their applications in differential equations
  • Ability to perform inverse Laplace transforms
NEXT STEPS
  • Study the properties of the Laplace transform, particularly for piecewise functions
  • Learn about the Heaviside step function and its role in differential equations
  • Practice solving second-order linear differential equations using the Laplace transform
  • Explore inverse Laplace transforms and their applications in engineering problems
USEFUL FOR

Students studying differential equations, particularly those focusing on Laplace transforms, as well as educators and tutors seeking to enhance their understanding of piecewise functions in mathematical modeling.

Destroxia
Messages
204
Reaction score
7

Homework Statement



##y'' + 2y' + 2y = h(t); y(0)=0, y'(0)=1,
h(t) = \begin{cases} t &, \pi \leq t < 2\pi \\
0 &, 3\pi \leq t < \infty \\
\end{cases}##



2. Homework Equations

The Attempt at a Solution



Take the Laplace of both sides:[/B]

##\mathcal{L}(y'' + 2y' + 2y) = \mathcal{L}(h(t)) ##

##s^2 F(s) - sf(0) - f'(0) + 2sF(s) - 2f(0) + 2F(s) = \mathcal{L}(h(t)) ##

##(s^2 + 2s +2)F(s) - 1 = \mathcal{L}(h(t)) = \int_{\pi}^{2\pi} te^{-st}##

##(s^2 + 2s +2)F(s) - 1 =\frac {\pi e^{-\pi s}} {s} + \frac {e^{-\pi s}} {s^2} - \frac {2\pi e^{-2\pi s}} {s} - \frac {e^{2\pi s}} {s^2} ##

Solve for F(s):

This is where I always screw up, If I even dare to add the 1 to the other side and divide by ##(s^2 + 2s + 2)## I get a ridiculously long expansion, that I don't even think is plausible for a practice problem, or any other problem.

I have no idea where to continue once I get to this point.
 
Physics news on Phys.org
RyanTAsher said:

Homework Statement



##y'' + 2y' + 2y = h(t); y(0)=0, y'(0)=1,
h(t) = \begin{cases} t &, \pi \leq t < 2\pi \\
0 &, 3\pi \leq t < \infty \\
\end{cases}##
2. Homework Equations

The Attempt at a Solution



Take the Laplace of both sides:[/B]

##\mathcal{L}(y'' + 2y' + 2y) = \mathcal{L}(h(t)) ##

##s^2 F(s) - sf(0) - f'(0) + 2sF(s) - 2f(0) + 2F(s) = \mathcal{L}(h(t)) ##

##(s^2 + 2s +2)F(s) - 1 = \mathcal{L}(h(t)) = \int_{\pi}^{2\pi} te^{-st}##

##(s^2 + 2s +2)F(s) - 1 =\frac {\pi e^{-\pi s}} {s} + \frac {e^{-\pi s}} {s^2} - \frac {2\pi e^{-2\pi s}} {s} - \frac {e^{2\pi s}} {s^2} ##

Solve for F(s):

This is where I always screw up, If I even dare to add the 1 to the other side and divide by ##(s^2 + 2s + 2)## I get a ridiculously long expansion, that I don't even think is plausible for a practice problem, or any other problem.

I have no idea where to continue once I get to this point.

It is actually not so bad, once you simplify things a bit. Start with
F(s) = \frac{e^{-\pi s}}{D_2(s)} + \frac{\pi e^{-\pi s}}{D_1(s)} - \frac{e^{-2 \pi s}}{D_2(s)} - \frac{2 \pi e^{-2 \pi s}}{D_1(s)} + \frac{1}{D_0(s)},
where ##D_2(s) = s^2 (s^2 + 2 s + 2), \: D_1(s) = s (s^2 + 2 s + 2), \; D_0(s) = s^2 + 2s + 2##.

It is convenient to first take the inverse Laplace transforms of the ##1/D_k(s)## functions:
\frac{1}{D_0(s)} \leftrightarrow e^{-t} \sin(t) \equiv e_0(t) \\<br /> \frac{1}{D_1(s)} = \frac{1}{s} \frac{1}{D_0(s)} \leftrightarrow \int_0^t e_0(\tau) \, d \tau = \frac{1}{2} - \frac{1}{2} e^{-t} (\sin(t) + \cos(t)) \equiv e_1(t) \\<br /> \frac{1}{D_2(s)} = \frac{1}{s} \frac{1}{D_1(s)} \leftrightarrow \int_0^t e_1 (\tau) \, d \tau = \frac{1}{2}(e^{-t} \cos(t) + t-1) \equiv e_2(t)

Now we have
F(t) = [e_2(t-\pi)+\pi e_1(t-\pi)]u(t-\pi) -[e_2(t-2 \pi) + 2 \pi e_1(t - 2 \pi)]u(t - 2 \pi) + e_0(t)

These are obtained using the following two standard properties, wherein below we have ##f(t) \leftrightarrow g(s)##:
\begin{array}{rl}1. &amp; \int_0^t f(\tau) d \tau \leftrightarrow \frac{1}{s} g(s) \\<br /> 2. &amp; f(t-a)u(t-a) \leftrightarrow e^{-as} g(s) \end{array}
 
Question: A clock's minute hand has length 4 and its hour hand has length 3. What is the distance between the tips at the moment when it is increasing most rapidly?(Putnam Exam Question) Answer: Making assumption that both the hands moves at constant angular velocities, the answer is ## \sqrt{7} .## But don't you think this assumption is somewhat doubtful and wrong?

Similar threads

  • · Replies 1 ·
Replies
1
Views
967
  • · Replies 7 ·
Replies
7
Views
2K
  • · Replies 11 ·
Replies
11
Views
3K
  • · Replies 6 ·
Replies
6
Views
2K
  • · Replies 1 ·
Replies
1
Views
1K
Replies
3
Views
2K
  • · Replies 3 ·
Replies
3
Views
3K
  • · Replies 6 ·
Replies
6
Views
1K
  • · Replies 1 ·
Replies
1
Views
1K
Replies
2
Views
2K